In most of this forest, the expected outbreak of tree-eating tussock moths should not be countered. After all, the mo...

joshuavt on June 12, 2018

Why A?

Could you please explain why A is the correct answer?

Replies
Create a free account to read and take part in forum discussions.

Already have an account? log in

joshuavt on June 13, 2018

Could you please explain why A is correct?

Christopher on June 15, 2018

@joshuavt, the conclusion of the argument is that in "most" of this forest, the moth outbreak should not be stopped. The premise is that moth outbreaks are beneficial to forests that are unnaturally crowded with immature trees. So to go from "moth outbreaks are beneficial in certain circumstances" to "we shouldn't stop this moth outbreak in most of this particular forest" requires evidence that "most of this particular forest" meets the conditions in which a moth outbreak would be beneficial. (A) provides this by saying that "more than half (most) of the forest is unnaturally crowded with immature trees" and would therefore benefit from the moth outbreak.


joshuavt on June 16, 2018

Thank you!

CrystalTudor250 on November 14, 2019

Can you explain to me why answer choice C would be wrong in this scenario?